Find a number such that $S(n)+P(n)=n$












0














Find a number $nin mathbb N$ such that $S(n)+P(n)=n$. Where $S(n)$ is sum of digits, and $P(n)$ is product of digits



I know that $9|n-S(n)$ ,since $n-S(n)=P(n)$ then $9|P(n)$ so at least one number must be nine or two digits $ldots33 ldots$ then I go with number $19,29,39,49,59,69,79,89,99$ for all this number $P(n)+S(n)=n$ but for number like 199 or 999 I just can not reach that number using formula, but I do not know how to prove it is not enough just to say that. And I do not know is there some other number can you help me?










share|cite|improve this question





























    0














    Find a number $nin mathbb N$ such that $S(n)+P(n)=n$. Where $S(n)$ is sum of digits, and $P(n)$ is product of digits



    I know that $9|n-S(n)$ ,since $n-S(n)=P(n)$ then $9|P(n)$ so at least one number must be nine or two digits $ldots33 ldots$ then I go with number $19,29,39,49,59,69,79,89,99$ for all this number $P(n)+S(n)=n$ but for number like 199 or 999 I just can not reach that number using formula, but I do not know how to prove it is not enough just to say that. And I do not know is there some other number can you help me?










    share|cite|improve this question



























      0












      0








      0







      Find a number $nin mathbb N$ such that $S(n)+P(n)=n$. Where $S(n)$ is sum of digits, and $P(n)$ is product of digits



      I know that $9|n-S(n)$ ,since $n-S(n)=P(n)$ then $9|P(n)$ so at least one number must be nine or two digits $ldots33 ldots$ then I go with number $19,29,39,49,59,69,79,89,99$ for all this number $P(n)+S(n)=n$ but for number like 199 or 999 I just can not reach that number using formula, but I do not know how to prove it is not enough just to say that. And I do not know is there some other number can you help me?










      share|cite|improve this question















      Find a number $nin mathbb N$ such that $S(n)+P(n)=n$. Where $S(n)$ is sum of digits, and $P(n)$ is product of digits



      I know that $9|n-S(n)$ ,since $n-S(n)=P(n)$ then $9|P(n)$ so at least one number must be nine or two digits $ldots33 ldots$ then I go with number $19,29,39,49,59,69,79,89,99$ for all this number $P(n)+S(n)=n$ but for number like 199 or 999 I just can not reach that number using formula, but I do not know how to prove it is not enough just to say that. And I do not know is there some other number can you help me?







      elementary-number-theory






      share|cite|improve this question















      share|cite|improve this question













      share|cite|improve this question




      share|cite|improve this question








      edited Nov 24 at 9:49









      greedoid

      37.7k114794




      37.7k114794










      asked Nov 24 at 5:13









      Marko Škorić

      70310




      70310






















          3 Answers
          3






          active

          oldest

          votes


















          1














          A couple of examples should suffice to show that we cannot have $n=S(n)+P(n)$ if $n$ has more than two digits:



          $$100a+10b+c=a+b+c+abcimplies c=9left({11over b}+{1over a} right)ge9cdot{11over10}gt9$$



          and



          $$1000a+100b+10c+d=a+b+c+d+abcdimplies d=9left({111over bc}+{11over ca}+{1over ab}right)ge9cdot{111over10cdot10}gt9$$



          In general, if $n=a_k10^k+cdots+a_0$ with $kgt1$, then $n=S(n)+P(n)$ implies the contradiction



          $$a_0ge9left((10^k-1)/9over a_{k-1}cdots a_1right)ge9left(1+10+cdots+10^{k-1}over10^{k-1} right)gt9$$



          (Note, for $k=1$, we just get $a_0ge9$, which is not a contradiction.)






          share|cite|improve this answer





























            1














            Hint: If a number $n$ is $k$ digits long and begins with the digit $d$, then



            $$S(n)leq 10(k-1)+d$$



            and



            $$P(n)leq d9^{k-1},$$



            so



            $$d10^{k-1}leq n=S(n)+P(n)leq dleft(9^{k-1}+1right)+10(k-1).$$



            $$dleft(10^{k-1}-9^{k-1}-1right)leq 10(k-1).$$



            $$10^{k-1}-9^{k-1}-1leq 10(k-1).$$



            Can you show that this has no solutions for $kgeq 4$, thus reducing it to the $k=3,d=1$ case? And then can you solve it from there using the same techniques you used in the body of your question?






            share|cite|improve this answer





























              1














              Hint:



              Write $$n= a_0+10a_1+10^2a_2+...+10^ka_k$$ where $a_kne 0$. Then $ngeq 10^k$, but $P(n)=a_0a_1...a_kleq 9^{k+1}$ and $S(n)leq 9(k+1)$. So we have $$10^kleq 9^{k+1}+9(k+1)$$



              Clearly $k$ can not be big in fact only several $k$ works here (with calculator I find $kleq 20$)...






              share|cite|improve this answer























                Your Answer





                StackExchange.ifUsing("editor", function () {
                return StackExchange.using("mathjaxEditing", function () {
                StackExchange.MarkdownEditor.creationCallbacks.add(function (editor, postfix) {
                StackExchange.mathjaxEditing.prepareWmdForMathJax(editor, postfix, [["$", "$"], ["\\(","\\)"]]);
                });
                });
                }, "mathjax-editing");

                StackExchange.ready(function() {
                var channelOptions = {
                tags: "".split(" "),
                id: "69"
                };
                initTagRenderer("".split(" "), "".split(" "), channelOptions);

                StackExchange.using("externalEditor", function() {
                // Have to fire editor after snippets, if snippets enabled
                if (StackExchange.settings.snippets.snippetsEnabled) {
                StackExchange.using("snippets", function() {
                createEditor();
                });
                }
                else {
                createEditor();
                }
                });

                function createEditor() {
                StackExchange.prepareEditor({
                heartbeatType: 'answer',
                autoActivateHeartbeat: false,
                convertImagesToLinks: true,
                noModals: true,
                showLowRepImageUploadWarning: true,
                reputationToPostImages: 10,
                bindNavPrevention: true,
                postfix: "",
                imageUploader: {
                brandingHtml: "Powered by u003ca class="icon-imgur-white" href="https://imgur.com/"u003eu003c/au003e",
                contentPolicyHtml: "User contributions licensed under u003ca href="https://creativecommons.org/licenses/by-sa/3.0/"u003ecc by-sa 3.0 with attribution requiredu003c/au003e u003ca href="https://stackoverflow.com/legal/content-policy"u003e(content policy)u003c/au003e",
                allowUrls: true
                },
                noCode: true, onDemand: true,
                discardSelector: ".discard-answer"
                ,immediatelyShowMarkdownHelp:true
                });


                }
                });














                draft saved

                draft discarded


















                StackExchange.ready(
                function () {
                StackExchange.openid.initPostLogin('.new-post-login', 'https%3a%2f%2fmath.stackexchange.com%2fquestions%2f3011211%2ffind-a-number-such-that-snpn-n%23new-answer', 'question_page');
                }
                );

                Post as a guest















                Required, but never shown

























                3 Answers
                3






                active

                oldest

                votes








                3 Answers
                3






                active

                oldest

                votes









                active

                oldest

                votes






                active

                oldest

                votes









                1














                A couple of examples should suffice to show that we cannot have $n=S(n)+P(n)$ if $n$ has more than two digits:



                $$100a+10b+c=a+b+c+abcimplies c=9left({11over b}+{1over a} right)ge9cdot{11over10}gt9$$



                and



                $$1000a+100b+10c+d=a+b+c+d+abcdimplies d=9left({111over bc}+{11over ca}+{1over ab}right)ge9cdot{111over10cdot10}gt9$$



                In general, if $n=a_k10^k+cdots+a_0$ with $kgt1$, then $n=S(n)+P(n)$ implies the contradiction



                $$a_0ge9left((10^k-1)/9over a_{k-1}cdots a_1right)ge9left(1+10+cdots+10^{k-1}over10^{k-1} right)gt9$$



                (Note, for $k=1$, we just get $a_0ge9$, which is not a contradiction.)






                share|cite|improve this answer


























                  1














                  A couple of examples should suffice to show that we cannot have $n=S(n)+P(n)$ if $n$ has more than two digits:



                  $$100a+10b+c=a+b+c+abcimplies c=9left({11over b}+{1over a} right)ge9cdot{11over10}gt9$$



                  and



                  $$1000a+100b+10c+d=a+b+c+d+abcdimplies d=9left({111over bc}+{11over ca}+{1over ab}right)ge9cdot{111over10cdot10}gt9$$



                  In general, if $n=a_k10^k+cdots+a_0$ with $kgt1$, then $n=S(n)+P(n)$ implies the contradiction



                  $$a_0ge9left((10^k-1)/9over a_{k-1}cdots a_1right)ge9left(1+10+cdots+10^{k-1}over10^{k-1} right)gt9$$



                  (Note, for $k=1$, we just get $a_0ge9$, which is not a contradiction.)






                  share|cite|improve this answer
























                    1












                    1








                    1






                    A couple of examples should suffice to show that we cannot have $n=S(n)+P(n)$ if $n$ has more than two digits:



                    $$100a+10b+c=a+b+c+abcimplies c=9left({11over b}+{1over a} right)ge9cdot{11over10}gt9$$



                    and



                    $$1000a+100b+10c+d=a+b+c+d+abcdimplies d=9left({111over bc}+{11over ca}+{1over ab}right)ge9cdot{111over10cdot10}gt9$$



                    In general, if $n=a_k10^k+cdots+a_0$ with $kgt1$, then $n=S(n)+P(n)$ implies the contradiction



                    $$a_0ge9left((10^k-1)/9over a_{k-1}cdots a_1right)ge9left(1+10+cdots+10^{k-1}over10^{k-1} right)gt9$$



                    (Note, for $k=1$, we just get $a_0ge9$, which is not a contradiction.)






                    share|cite|improve this answer












                    A couple of examples should suffice to show that we cannot have $n=S(n)+P(n)$ if $n$ has more than two digits:



                    $$100a+10b+c=a+b+c+abcimplies c=9left({11over b}+{1over a} right)ge9cdot{11over10}gt9$$



                    and



                    $$1000a+100b+10c+d=a+b+c+d+abcdimplies d=9left({111over bc}+{11over ca}+{1over ab}right)ge9cdot{111over10cdot10}gt9$$



                    In general, if $n=a_k10^k+cdots+a_0$ with $kgt1$, then $n=S(n)+P(n)$ implies the contradiction



                    $$a_0ge9left((10^k-1)/9over a_{k-1}cdots a_1right)ge9left(1+10+cdots+10^{k-1}over10^{k-1} right)gt9$$



                    (Note, for $k=1$, we just get $a_0ge9$, which is not a contradiction.)







                    share|cite|improve this answer












                    share|cite|improve this answer



                    share|cite|improve this answer










                    answered Nov 24 at 12:04









                    Barry Cipra

                    59k653123




                    59k653123























                        1














                        Hint: If a number $n$ is $k$ digits long and begins with the digit $d$, then



                        $$S(n)leq 10(k-1)+d$$



                        and



                        $$P(n)leq d9^{k-1},$$



                        so



                        $$d10^{k-1}leq n=S(n)+P(n)leq dleft(9^{k-1}+1right)+10(k-1).$$



                        $$dleft(10^{k-1}-9^{k-1}-1right)leq 10(k-1).$$



                        $$10^{k-1}-9^{k-1}-1leq 10(k-1).$$



                        Can you show that this has no solutions for $kgeq 4$, thus reducing it to the $k=3,d=1$ case? And then can you solve it from there using the same techniques you used in the body of your question?






                        share|cite|improve this answer


























                          1














                          Hint: If a number $n$ is $k$ digits long and begins with the digit $d$, then



                          $$S(n)leq 10(k-1)+d$$



                          and



                          $$P(n)leq d9^{k-1},$$



                          so



                          $$d10^{k-1}leq n=S(n)+P(n)leq dleft(9^{k-1}+1right)+10(k-1).$$



                          $$dleft(10^{k-1}-9^{k-1}-1right)leq 10(k-1).$$



                          $$10^{k-1}-9^{k-1}-1leq 10(k-1).$$



                          Can you show that this has no solutions for $kgeq 4$, thus reducing it to the $k=3,d=1$ case? And then can you solve it from there using the same techniques you used in the body of your question?






                          share|cite|improve this answer
























                            1












                            1








                            1






                            Hint: If a number $n$ is $k$ digits long and begins with the digit $d$, then



                            $$S(n)leq 10(k-1)+d$$



                            and



                            $$P(n)leq d9^{k-1},$$



                            so



                            $$d10^{k-1}leq n=S(n)+P(n)leq dleft(9^{k-1}+1right)+10(k-1).$$



                            $$dleft(10^{k-1}-9^{k-1}-1right)leq 10(k-1).$$



                            $$10^{k-1}-9^{k-1}-1leq 10(k-1).$$



                            Can you show that this has no solutions for $kgeq 4$, thus reducing it to the $k=3,d=1$ case? And then can you solve it from there using the same techniques you used in the body of your question?






                            share|cite|improve this answer












                            Hint: If a number $n$ is $k$ digits long and begins with the digit $d$, then



                            $$S(n)leq 10(k-1)+d$$



                            and



                            $$P(n)leq d9^{k-1},$$



                            so



                            $$d10^{k-1}leq n=S(n)+P(n)leq dleft(9^{k-1}+1right)+10(k-1).$$



                            $$dleft(10^{k-1}-9^{k-1}-1right)leq 10(k-1).$$



                            $$10^{k-1}-9^{k-1}-1leq 10(k-1).$$



                            Can you show that this has no solutions for $kgeq 4$, thus reducing it to the $k=3,d=1$ case? And then can you solve it from there using the same techniques you used in the body of your question?







                            share|cite|improve this answer












                            share|cite|improve this answer



                            share|cite|improve this answer










                            answered Nov 24 at 5:22









                            Carl Schildkraut

                            11.1k11441




                            11.1k11441























                                1














                                Hint:



                                Write $$n= a_0+10a_1+10^2a_2+...+10^ka_k$$ where $a_kne 0$. Then $ngeq 10^k$, but $P(n)=a_0a_1...a_kleq 9^{k+1}$ and $S(n)leq 9(k+1)$. So we have $$10^kleq 9^{k+1}+9(k+1)$$



                                Clearly $k$ can not be big in fact only several $k$ works here (with calculator I find $kleq 20$)...






                                share|cite|improve this answer




























                                  1














                                  Hint:



                                  Write $$n= a_0+10a_1+10^2a_2+...+10^ka_k$$ where $a_kne 0$. Then $ngeq 10^k$, but $P(n)=a_0a_1...a_kleq 9^{k+1}$ and $S(n)leq 9(k+1)$. So we have $$10^kleq 9^{k+1}+9(k+1)$$



                                  Clearly $k$ can not be big in fact only several $k$ works here (with calculator I find $kleq 20$)...






                                  share|cite|improve this answer


























                                    1












                                    1








                                    1






                                    Hint:



                                    Write $$n= a_0+10a_1+10^2a_2+...+10^ka_k$$ where $a_kne 0$. Then $ngeq 10^k$, but $P(n)=a_0a_1...a_kleq 9^{k+1}$ and $S(n)leq 9(k+1)$. So we have $$10^kleq 9^{k+1}+9(k+1)$$



                                    Clearly $k$ can not be big in fact only several $k$ works here (with calculator I find $kleq 20$)...






                                    share|cite|improve this answer














                                    Hint:



                                    Write $$n= a_0+10a_1+10^2a_2+...+10^ka_k$$ where $a_kne 0$. Then $ngeq 10^k$, but $P(n)=a_0a_1...a_kleq 9^{k+1}$ and $S(n)leq 9(k+1)$. So we have $$10^kleq 9^{k+1}+9(k+1)$$



                                    Clearly $k$ can not be big in fact only several $k$ works here (with calculator I find $kleq 20$)...







                                    share|cite|improve this answer














                                    share|cite|improve this answer



                                    share|cite|improve this answer








                                    edited Nov 24 at 10:47

























                                    answered Nov 24 at 9:48









                                    greedoid

                                    37.7k114794




                                    37.7k114794






























                                        draft saved

                                        draft discarded




















































                                        Thanks for contributing an answer to Mathematics Stack Exchange!


                                        • Please be sure to answer the question. Provide details and share your research!

                                        But avoid



                                        • Asking for help, clarification, or responding to other answers.

                                        • Making statements based on opinion; back them up with references or personal experience.


                                        Use MathJax to format equations. MathJax reference.


                                        To learn more, see our tips on writing great answers.





                                        Some of your past answers have not been well-received, and you're in danger of being blocked from answering.


                                        Please pay close attention to the following guidance:


                                        • Please be sure to answer the question. Provide details and share your research!

                                        But avoid



                                        • Asking for help, clarification, or responding to other answers.

                                        • Making statements based on opinion; back them up with references or personal experience.


                                        To learn more, see our tips on writing great answers.




                                        draft saved


                                        draft discarded














                                        StackExchange.ready(
                                        function () {
                                        StackExchange.openid.initPostLogin('.new-post-login', 'https%3a%2f%2fmath.stackexchange.com%2fquestions%2f3011211%2ffind-a-number-such-that-snpn-n%23new-answer', 'question_page');
                                        }
                                        );

                                        Post as a guest















                                        Required, but never shown





















































                                        Required, but never shown














                                        Required, but never shown












                                        Required, but never shown







                                        Required, but never shown

































                                        Required, but never shown














                                        Required, but never shown












                                        Required, but never shown







                                        Required, but never shown







                                        Popular posts from this blog

                                        Plaza Victoria

                                        In PowerPoint, is there a keyboard shortcut for bulleted / numbered list?

                                        How to put 3 figures in Latex with 2 figures side by side and 1 below these side by side images but in...